Proving that $sum_{n = 1}^{infty} frac{1}{n^s} = prod_{k=1}^{infty}frac{1}{1-p_k^{-s}}$












0












$begingroup$


Could you please have a look at my solution of the following exercise?




Show that for $s>1$ holds
$$sum_{n = 1}^{infty} frac{1}{n^s} = prod_{k=1}^{infty}frac{1}{1-p_k^{-s}}$$



where $(p_k)_k$ is the increasing sequence of primes.




I came up with the following solution:



At first we note that



$$prod_{k=1}^{infty}frac{1}{1-p_k^{-s}} = prod_{k=1}^{infty} frac{p_k^s}{p_k^s-1} = prod_{k=1}^{infty} biggl( sum_{j=0}^{infty} frac{1}{p_k^{s cdot j}}biggr)$$



Now we show both $ge$ and $le$:



$ge:$



$$prod_{k=1}^{N}frac{1}{1-p_k^{-s}} = prod_{k=1}^{N} frac{p_k^s}{p_k^s-1} $$



To make things easier we take care that the exponents are natural numbers



$$le prod_{k=1}^{N} frac{p_k^{lceil s rceil}}{p_k^{lfloor s rfloor} -1}$$



For fitting natural numbers $A,B$ we may write



$$ = frac{A}{B} = sum_{n=1}^{A} frac{1}{B} $$



And since $A ge B$



$$le sum_{n=1}^{A} frac{1}{n}$$



$le:$



We make the observation that in the result of the multiplication



$$biggl( sum_{i=0}^{a} frac{1}{p_1^i} biggr) cdot biggl( sum_{j=0}^{b} frac{1}{p_2^j}biggr) = sum_{i=0}^{a} sum_{j=0}^{b} frac{1}{p_1^i cdot p_2^j} $$



each combination of powers of $p_1$ and $p_2$ up to $p_1^a$ and $p_2^b$ appears exactly once in the denominators on the right side above. By generalising this and remembering about the fundametal theorem of arithmetic we obtain:



$$sum_{n = 1}^{N} frac{1}{n^s} le prod_{k=1}^{A} biggl( sum_{j=0}^{B_k} frac{1}{p_k^{s cdot j}}biggr) $$



for some natural numbers $A$ and $B$ and further:



$$le prod_{k=1}^{A} biggl( sum_{j=0}^{infty} frac{1}{p_k^{s cdot j}}biggr) = prod_{k=1}^{A} frac{p_k^s}{p_k^s-1} le prod_{k=1}^{infty} frac{p_k^s}{p_k^s-1}$$



Is this correct and if yes, could you tell me where exactly we need that $s>1$; I do not see why this is required.










share|cite|improve this question











$endgroup$








  • 1




    $begingroup$
    Let $A_K$ be the set of integers whose prime factors are all $le p_K$. Then for $s > 0$, $prod_{k=1}^K frac{1}{1-p_k^{-s}} = prod_{k=1}^K (1+ sum_{j=1}^infty (p_k^j)^{-s}) = sum_{n in A_K} n^{-s}$. For $s > 1$ then $sum_{n=1}^infty n^{-s} - prod_{k=1}^K frac{1}{1-p_k^{-s}}=sum_{n not in A_K}n^{-s}le sum_{n=P_K+1}^infty n^{-s}$ whence $lim_{K to infty}sum_{n=1}^infty n^{-s}-prod_{k=1}^Kfrac{1}{1-p_k^{-s}} = 0$. For $sin (0,1]$ then $lim_{K to infty}prod_{k=1}^Kfrac{1}{1-p_k^{-s}}=lim_{K to infty}sum_{nin A_K}n^{-s}=sum_{n=1}^infty n^{-s}= infty$
    $endgroup$
    – reuns
    Jan 6 at 17:36








  • 2




    $begingroup$
    "could you tell me where exactly we need that $s>1$" Actually, we do not. When $sleqslant1$, the identity in the question holds as well, but it reads $infty=infty$.
    $endgroup$
    – Did
    Jan 6 at 17:54








  • 1




    $begingroup$
    See math.stackexchange.com/questions/427910/…?
    $endgroup$
    – Robert Z
    Feb 14 at 11:45
















0












$begingroup$


Could you please have a look at my solution of the following exercise?




Show that for $s>1$ holds
$$sum_{n = 1}^{infty} frac{1}{n^s} = prod_{k=1}^{infty}frac{1}{1-p_k^{-s}}$$



where $(p_k)_k$ is the increasing sequence of primes.




I came up with the following solution:



At first we note that



$$prod_{k=1}^{infty}frac{1}{1-p_k^{-s}} = prod_{k=1}^{infty} frac{p_k^s}{p_k^s-1} = prod_{k=1}^{infty} biggl( sum_{j=0}^{infty} frac{1}{p_k^{s cdot j}}biggr)$$



Now we show both $ge$ and $le$:



$ge:$



$$prod_{k=1}^{N}frac{1}{1-p_k^{-s}} = prod_{k=1}^{N} frac{p_k^s}{p_k^s-1} $$



To make things easier we take care that the exponents are natural numbers



$$le prod_{k=1}^{N} frac{p_k^{lceil s rceil}}{p_k^{lfloor s rfloor} -1}$$



For fitting natural numbers $A,B$ we may write



$$ = frac{A}{B} = sum_{n=1}^{A} frac{1}{B} $$



And since $A ge B$



$$le sum_{n=1}^{A} frac{1}{n}$$



$le:$



We make the observation that in the result of the multiplication



$$biggl( sum_{i=0}^{a} frac{1}{p_1^i} biggr) cdot biggl( sum_{j=0}^{b} frac{1}{p_2^j}biggr) = sum_{i=0}^{a} sum_{j=0}^{b} frac{1}{p_1^i cdot p_2^j} $$



each combination of powers of $p_1$ and $p_2$ up to $p_1^a$ and $p_2^b$ appears exactly once in the denominators on the right side above. By generalising this and remembering about the fundametal theorem of arithmetic we obtain:



$$sum_{n = 1}^{N} frac{1}{n^s} le prod_{k=1}^{A} biggl( sum_{j=0}^{B_k} frac{1}{p_k^{s cdot j}}biggr) $$



for some natural numbers $A$ and $B$ and further:



$$le prod_{k=1}^{A} biggl( sum_{j=0}^{infty} frac{1}{p_k^{s cdot j}}biggr) = prod_{k=1}^{A} frac{p_k^s}{p_k^s-1} le prod_{k=1}^{infty} frac{p_k^s}{p_k^s-1}$$



Is this correct and if yes, could you tell me where exactly we need that $s>1$; I do not see why this is required.










share|cite|improve this question











$endgroup$








  • 1




    $begingroup$
    Let $A_K$ be the set of integers whose prime factors are all $le p_K$. Then for $s > 0$, $prod_{k=1}^K frac{1}{1-p_k^{-s}} = prod_{k=1}^K (1+ sum_{j=1}^infty (p_k^j)^{-s}) = sum_{n in A_K} n^{-s}$. For $s > 1$ then $sum_{n=1}^infty n^{-s} - prod_{k=1}^K frac{1}{1-p_k^{-s}}=sum_{n not in A_K}n^{-s}le sum_{n=P_K+1}^infty n^{-s}$ whence $lim_{K to infty}sum_{n=1}^infty n^{-s}-prod_{k=1}^Kfrac{1}{1-p_k^{-s}} = 0$. For $sin (0,1]$ then $lim_{K to infty}prod_{k=1}^Kfrac{1}{1-p_k^{-s}}=lim_{K to infty}sum_{nin A_K}n^{-s}=sum_{n=1}^infty n^{-s}= infty$
    $endgroup$
    – reuns
    Jan 6 at 17:36








  • 2




    $begingroup$
    "could you tell me where exactly we need that $s>1$" Actually, we do not. When $sleqslant1$, the identity in the question holds as well, but it reads $infty=infty$.
    $endgroup$
    – Did
    Jan 6 at 17:54








  • 1




    $begingroup$
    See math.stackexchange.com/questions/427910/…?
    $endgroup$
    – Robert Z
    Feb 14 at 11:45














0












0








0





$begingroup$


Could you please have a look at my solution of the following exercise?




Show that for $s>1$ holds
$$sum_{n = 1}^{infty} frac{1}{n^s} = prod_{k=1}^{infty}frac{1}{1-p_k^{-s}}$$



where $(p_k)_k$ is the increasing sequence of primes.




I came up with the following solution:



At first we note that



$$prod_{k=1}^{infty}frac{1}{1-p_k^{-s}} = prod_{k=1}^{infty} frac{p_k^s}{p_k^s-1} = prod_{k=1}^{infty} biggl( sum_{j=0}^{infty} frac{1}{p_k^{s cdot j}}biggr)$$



Now we show both $ge$ and $le$:



$ge:$



$$prod_{k=1}^{N}frac{1}{1-p_k^{-s}} = prod_{k=1}^{N} frac{p_k^s}{p_k^s-1} $$



To make things easier we take care that the exponents are natural numbers



$$le prod_{k=1}^{N} frac{p_k^{lceil s rceil}}{p_k^{lfloor s rfloor} -1}$$



For fitting natural numbers $A,B$ we may write



$$ = frac{A}{B} = sum_{n=1}^{A} frac{1}{B} $$



And since $A ge B$



$$le sum_{n=1}^{A} frac{1}{n}$$



$le:$



We make the observation that in the result of the multiplication



$$biggl( sum_{i=0}^{a} frac{1}{p_1^i} biggr) cdot biggl( sum_{j=0}^{b} frac{1}{p_2^j}biggr) = sum_{i=0}^{a} sum_{j=0}^{b} frac{1}{p_1^i cdot p_2^j} $$



each combination of powers of $p_1$ and $p_2$ up to $p_1^a$ and $p_2^b$ appears exactly once in the denominators on the right side above. By generalising this and remembering about the fundametal theorem of arithmetic we obtain:



$$sum_{n = 1}^{N} frac{1}{n^s} le prod_{k=1}^{A} biggl( sum_{j=0}^{B_k} frac{1}{p_k^{s cdot j}}biggr) $$



for some natural numbers $A$ and $B$ and further:



$$le prod_{k=1}^{A} biggl( sum_{j=0}^{infty} frac{1}{p_k^{s cdot j}}biggr) = prod_{k=1}^{A} frac{p_k^s}{p_k^s-1} le prod_{k=1}^{infty} frac{p_k^s}{p_k^s-1}$$



Is this correct and if yes, could you tell me where exactly we need that $s>1$; I do not see why this is required.










share|cite|improve this question











$endgroup$




Could you please have a look at my solution of the following exercise?




Show that for $s>1$ holds
$$sum_{n = 1}^{infty} frac{1}{n^s} = prod_{k=1}^{infty}frac{1}{1-p_k^{-s}}$$



where $(p_k)_k$ is the increasing sequence of primes.




I came up with the following solution:



At first we note that



$$prod_{k=1}^{infty}frac{1}{1-p_k^{-s}} = prod_{k=1}^{infty} frac{p_k^s}{p_k^s-1} = prod_{k=1}^{infty} biggl( sum_{j=0}^{infty} frac{1}{p_k^{s cdot j}}biggr)$$



Now we show both $ge$ and $le$:



$ge:$



$$prod_{k=1}^{N}frac{1}{1-p_k^{-s}} = prod_{k=1}^{N} frac{p_k^s}{p_k^s-1} $$



To make things easier we take care that the exponents are natural numbers



$$le prod_{k=1}^{N} frac{p_k^{lceil s rceil}}{p_k^{lfloor s rfloor} -1}$$



For fitting natural numbers $A,B$ we may write



$$ = frac{A}{B} = sum_{n=1}^{A} frac{1}{B} $$



And since $A ge B$



$$le sum_{n=1}^{A} frac{1}{n}$$



$le:$



We make the observation that in the result of the multiplication



$$biggl( sum_{i=0}^{a} frac{1}{p_1^i} biggr) cdot biggl( sum_{j=0}^{b} frac{1}{p_2^j}biggr) = sum_{i=0}^{a} sum_{j=0}^{b} frac{1}{p_1^i cdot p_2^j} $$



each combination of powers of $p_1$ and $p_2$ up to $p_1^a$ and $p_2^b$ appears exactly once in the denominators on the right side above. By generalising this and remembering about the fundametal theorem of arithmetic we obtain:



$$sum_{n = 1}^{N} frac{1}{n^s} le prod_{k=1}^{A} biggl( sum_{j=0}^{B_k} frac{1}{p_k^{s cdot j}}biggr) $$



for some natural numbers $A$ and $B$ and further:



$$le prod_{k=1}^{A} biggl( sum_{j=0}^{infty} frac{1}{p_k^{s cdot j}}biggr) = prod_{k=1}^{A} frac{p_k^s}{p_k^s-1} le prod_{k=1}^{infty} frac{p_k^s}{p_k^s-1}$$



Is this correct and if yes, could you tell me where exactly we need that $s>1$; I do not see why this is required.







sequences-and-series number-theory






share|cite|improve this question















share|cite|improve this question













share|cite|improve this question




share|cite|improve this question








edited Jan 30 at 15:38







3nondatur

















asked Jan 6 at 16:47









3nondatur3nondatur

406112




406112








  • 1




    $begingroup$
    Let $A_K$ be the set of integers whose prime factors are all $le p_K$. Then for $s > 0$, $prod_{k=1}^K frac{1}{1-p_k^{-s}} = prod_{k=1}^K (1+ sum_{j=1}^infty (p_k^j)^{-s}) = sum_{n in A_K} n^{-s}$. For $s > 1$ then $sum_{n=1}^infty n^{-s} - prod_{k=1}^K frac{1}{1-p_k^{-s}}=sum_{n not in A_K}n^{-s}le sum_{n=P_K+1}^infty n^{-s}$ whence $lim_{K to infty}sum_{n=1}^infty n^{-s}-prod_{k=1}^Kfrac{1}{1-p_k^{-s}} = 0$. For $sin (0,1]$ then $lim_{K to infty}prod_{k=1}^Kfrac{1}{1-p_k^{-s}}=lim_{K to infty}sum_{nin A_K}n^{-s}=sum_{n=1}^infty n^{-s}= infty$
    $endgroup$
    – reuns
    Jan 6 at 17:36








  • 2




    $begingroup$
    "could you tell me where exactly we need that $s>1$" Actually, we do not. When $sleqslant1$, the identity in the question holds as well, but it reads $infty=infty$.
    $endgroup$
    – Did
    Jan 6 at 17:54








  • 1




    $begingroup$
    See math.stackexchange.com/questions/427910/…?
    $endgroup$
    – Robert Z
    Feb 14 at 11:45














  • 1




    $begingroup$
    Let $A_K$ be the set of integers whose prime factors are all $le p_K$. Then for $s > 0$, $prod_{k=1}^K frac{1}{1-p_k^{-s}} = prod_{k=1}^K (1+ sum_{j=1}^infty (p_k^j)^{-s}) = sum_{n in A_K} n^{-s}$. For $s > 1$ then $sum_{n=1}^infty n^{-s} - prod_{k=1}^K frac{1}{1-p_k^{-s}}=sum_{n not in A_K}n^{-s}le sum_{n=P_K+1}^infty n^{-s}$ whence $lim_{K to infty}sum_{n=1}^infty n^{-s}-prod_{k=1}^Kfrac{1}{1-p_k^{-s}} = 0$. For $sin (0,1]$ then $lim_{K to infty}prod_{k=1}^Kfrac{1}{1-p_k^{-s}}=lim_{K to infty}sum_{nin A_K}n^{-s}=sum_{n=1}^infty n^{-s}= infty$
    $endgroup$
    – reuns
    Jan 6 at 17:36








  • 2




    $begingroup$
    "could you tell me where exactly we need that $s>1$" Actually, we do not. When $sleqslant1$, the identity in the question holds as well, but it reads $infty=infty$.
    $endgroup$
    – Did
    Jan 6 at 17:54








  • 1




    $begingroup$
    See math.stackexchange.com/questions/427910/…?
    $endgroup$
    – Robert Z
    Feb 14 at 11:45








1




1




$begingroup$
Let $A_K$ be the set of integers whose prime factors are all $le p_K$. Then for $s > 0$, $prod_{k=1}^K frac{1}{1-p_k^{-s}} = prod_{k=1}^K (1+ sum_{j=1}^infty (p_k^j)^{-s}) = sum_{n in A_K} n^{-s}$. For $s > 1$ then $sum_{n=1}^infty n^{-s} - prod_{k=1}^K frac{1}{1-p_k^{-s}}=sum_{n not in A_K}n^{-s}le sum_{n=P_K+1}^infty n^{-s}$ whence $lim_{K to infty}sum_{n=1}^infty n^{-s}-prod_{k=1}^Kfrac{1}{1-p_k^{-s}} = 0$. For $sin (0,1]$ then $lim_{K to infty}prod_{k=1}^Kfrac{1}{1-p_k^{-s}}=lim_{K to infty}sum_{nin A_K}n^{-s}=sum_{n=1}^infty n^{-s}= infty$
$endgroup$
– reuns
Jan 6 at 17:36






$begingroup$
Let $A_K$ be the set of integers whose prime factors are all $le p_K$. Then for $s > 0$, $prod_{k=1}^K frac{1}{1-p_k^{-s}} = prod_{k=1}^K (1+ sum_{j=1}^infty (p_k^j)^{-s}) = sum_{n in A_K} n^{-s}$. For $s > 1$ then $sum_{n=1}^infty n^{-s} - prod_{k=1}^K frac{1}{1-p_k^{-s}}=sum_{n not in A_K}n^{-s}le sum_{n=P_K+1}^infty n^{-s}$ whence $lim_{K to infty}sum_{n=1}^infty n^{-s}-prod_{k=1}^Kfrac{1}{1-p_k^{-s}} = 0$. For $sin (0,1]$ then $lim_{K to infty}prod_{k=1}^Kfrac{1}{1-p_k^{-s}}=lim_{K to infty}sum_{nin A_K}n^{-s}=sum_{n=1}^infty n^{-s}= infty$
$endgroup$
– reuns
Jan 6 at 17:36






2




2




$begingroup$
"could you tell me where exactly we need that $s>1$" Actually, we do not. When $sleqslant1$, the identity in the question holds as well, but it reads $infty=infty$.
$endgroup$
– Did
Jan 6 at 17:54






$begingroup$
"could you tell me where exactly we need that $s>1$" Actually, we do not. When $sleqslant1$, the identity in the question holds as well, but it reads $infty=infty$.
$endgroup$
– Did
Jan 6 at 17:54






1




1




$begingroup$
See math.stackexchange.com/questions/427910/…?
$endgroup$
– Robert Z
Feb 14 at 11:45




$begingroup$
See math.stackexchange.com/questions/427910/…?
$endgroup$
– Robert Z
Feb 14 at 11:45










0






active

oldest

votes












Your Answer





StackExchange.ifUsing("editor", function () {
return StackExchange.using("mathjaxEditing", function () {
StackExchange.MarkdownEditor.creationCallbacks.add(function (editor, postfix) {
StackExchange.mathjaxEditing.prepareWmdForMathJax(editor, postfix, [["$", "$"], ["\\(","\\)"]]);
});
});
}, "mathjax-editing");

StackExchange.ready(function() {
var channelOptions = {
tags: "".split(" "),
id: "69"
};
initTagRenderer("".split(" "), "".split(" "), channelOptions);

StackExchange.using("externalEditor", function() {
// Have to fire editor after snippets, if snippets enabled
if (StackExchange.settings.snippets.snippetsEnabled) {
StackExchange.using("snippets", function() {
createEditor();
});
}
else {
createEditor();
}
});

function createEditor() {
StackExchange.prepareEditor({
heartbeatType: 'answer',
autoActivateHeartbeat: false,
convertImagesToLinks: true,
noModals: true,
showLowRepImageUploadWarning: true,
reputationToPostImages: 10,
bindNavPrevention: true,
postfix: "",
imageUploader: {
brandingHtml: "Powered by u003ca class="icon-imgur-white" href="https://imgur.com/"u003eu003c/au003e",
contentPolicyHtml: "User contributions licensed under u003ca href="https://creativecommons.org/licenses/by-sa/3.0/"u003ecc by-sa 3.0 with attribution requiredu003c/au003e u003ca href="https://stackoverflow.com/legal/content-policy"u003e(content policy)u003c/au003e",
allowUrls: true
},
noCode: true, onDemand: true,
discardSelector: ".discard-answer"
,immediatelyShowMarkdownHelp:true
});


}
});














draft saved

draft discarded


















StackExchange.ready(
function () {
StackExchange.openid.initPostLogin('.new-post-login', 'https%3a%2f%2fmath.stackexchange.com%2fquestions%2f3064099%2fproving-that-sum-n-1-infty-frac1ns-prod-k-1-infty-frac1%23new-answer', 'question_page');
}
);

Post as a guest















Required, but never shown

























0






active

oldest

votes








0






active

oldest

votes









active

oldest

votes






active

oldest

votes
















draft saved

draft discarded




















































Thanks for contributing an answer to Mathematics Stack Exchange!


  • Please be sure to answer the question. Provide details and share your research!

But avoid



  • Asking for help, clarification, or responding to other answers.

  • Making statements based on opinion; back them up with references or personal experience.


Use MathJax to format equations. MathJax reference.


To learn more, see our tips on writing great answers.




draft saved


draft discarded














StackExchange.ready(
function () {
StackExchange.openid.initPostLogin('.new-post-login', 'https%3a%2f%2fmath.stackexchange.com%2fquestions%2f3064099%2fproving-that-sum-n-1-infty-frac1ns-prod-k-1-infty-frac1%23new-answer', 'question_page');
}
);

Post as a guest















Required, but never shown





















































Required, but never shown














Required, but never shown












Required, but never shown







Required, but never shown

































Required, but never shown














Required, but never shown












Required, but never shown







Required, but never shown







Popular posts from this blog

To store a contact into the json file from server.js file using a class in NodeJS

Redirect URL with Chrome Remote Debugging Android Devices

Dieringhausen